Mock Exam Practice Flashcards
A 43-year-old man has a 10-year history of generalized anxiety disorder (GAD). He has been unable to tolerate paroxetine, escitalopram, or venlafaxine. His medical history is also significant for type 2 diabetes with painful peripheral neuropathy, dyslipidemia, and hypertension. He takes metformin, lisinopril, rosuvastatin, empagliflozin, and alprazolam. Which medication is best for long-term control of this patient’s anxiety symptoms?
A Buspirone. B Duloxetine. C Pregabalin. D Sertraline.
C - Pregablin. Is a 1st line treatment for GAD and is good for patients who can’t tolerate SSRIs or SNRIs. Duloxetine is wrong because patient has a history of intolerance for Several SSRIs
A 27-year-old woman with Crohn’s disease presents to the infusion center pharmacy with a new prescription for vedolizumab infusion. She has no other significant medical history. She states that she has tried several other medications in the past with some success; most recently, she has been treated with infliximab infusion but has had reduced efficacy with the past several infusions. Which education point is most important to share with your patient?
A Vedolizumab is an integrin inhibitor associated with a risk of rare but fatal progressive multifocal leukoencephalopathy (PML). B Biologic agents increase the risk of developing congestive heart failure and should be avoided in young patients. C Because you had loss of efficacy to infliximab, vedolizumab is not likely to be effective for you. D Vedolizumab is administered by intravenous infusion, requiring you to come to the infusion center for each dose.
D - Because it’s a MAB
J.A. is a 64-year-old man (height 70 inches, weight 90 kg) who is currently undergoing aortic valve replacement with a mechanical heart valve. His medical history includes hypertension, dyslipidemia, and generalized anxiety. His current medications include carvedilol 25 mg twice daily, lisinopril 10 mg once daily, atorvastatin 20 mg once daily, and escitalopram 10 mg daily. His vital signs and laboratory values are all within normal limits. Which is the most appropriate antithrombotic regimen for J.A.?
A Apixaban 5 mg twice daily. B Adjusted-dose warfarin to an INR of 2.5–3.5 with aspirin 81 mg daily. C Aspirin 325 mg daily. D Adjusted-dose warfarin to an INR of 2.0–3.0 with aspirin 81 mg daily.
D - A and C are wrong because they are CI’d in patients with valvular disease. B is for mitral valves not aortic.
A randomized controlled trial compared a new anticoagulant (drug A) with warfarin for the treatment of venous thromboembolism. One of the safety end points was the incidence of clinically significant bleeding, which was 2.3% in the drug A group compared with 4.1% in the warfarin group. The 95% CI for the difference was -1.9% to 2.4%. Which conclusion is most appropriate?
A Drug A was superior to warfarin with respect to bleeding complications. B Superiority of drug A over warfarin could not be established. C Warfarin was inferior to drug A with respect to bleeding complications. D No conclusion can be drawn because p values are unavailable.
B - The CI includes 0, thus no statistically significant difference.
A 67-year-old woman presents to the ED with septic shock, presumably caused by a UTI. Her medical history is significant for hypertension, hyperlipidemia, and recurrent Escherichia coli UTIs. Pertinent vital signs include blood pressure 82/44 mm Hg, heart rate 106 beats/minute, respiratory rate 28 breaths/minute, and temperature 103.6ºF (39.8ºC). Pertinent laboratory values include WBC 24 × 103 cells/mm3, Na 148 mEq/L, K 3.8 mEq/L, Cl 110 mEq/L, HCO3 19 mEq/L, SCr 1.9 mg/dL, and lactate 4.1 mmol/L. On physical examination, the patient is confused and lethargic with a Glasgow Coma Scale score of 12, and she weighs 68 kg. She is given 2 L of lactated Ringer solution, cultures are sent, and broad-spectrum antibiotics are being administered.
Question 5
An arterial blood gas shows pH 7.28, Paco2 32 mm Hg, and Pao2 56 mm Hg. Which most accurately describes her acid-base status?
A Anion gap metabolic acidosis. B Non-anion gap metabolic acidosis. C Respiratory acidosis. D Respiratory alkalosis.
A - patient has a low PH and therefore has an acidosis (D is incorrect). Anion gap is 148 - (110+19) = 19 which is above 12 (the upper limit of normal) and therefore shows that anions are in the blood (B is incorrect). Paco2 is low indicating respiratory compensation for the primary metabolic disturbance (c is incorrect).
A 67-year-old woman presents to the ED with septic shock, presumably caused by a UTI. Her medical history is significant for hypertension, hyperlipidemia, and recurrent Escherichia coli UTIs. Pertinent vital signs include blood pressure 82/44 mm Hg, heart rate 106 beats/minute, respiratory rate 28 breaths/minute, and temperature 103.6°F (39.8°C). Pertinent laboratory values include WBC 24 × 103 cells/mm3, Na 148 mEq/L, K 3.8 mEq/L, Cl 110 mEq/L, HCO3 19 mEq/L, SCr 1.9 mg/dL, and lactate 4.1 mmol/L. On physical examination, the patient is confused and lethargic with a Glasgow Coma Scale score of 12, and she weighs 68 kg. She is given 2 L of lactated Ringer solution, cultures are sent, and broad-spectrum antibiotics are being administered.
Question 6
After administration of crystalloids, the patient’s blood pressure is 86/46 mm Hg. Which is the most appropriate intervention to implement next?
A Initiate dobutamine 2 mcg/kg/minute because she has cardiogenic shock. B Initiate vasopressin 0.03 units/minute because she has distributive shock. C Initiate norepinephrine 0.05 mcg/kg/minute because she has distributive shock. D No additional intervention is needed because the patient improved with fluid administration.
C - initiate norepinephrine 0.05 mcg/kg/minute because she has distributive shock, is correct. The patient is in septic shock and remains hypotensive despite adequate fluid administration (Answer D is incorrect). Septic shock is a type of distributive shock, and dobutamine, used for cardiogenic shock, is not indicated at this time (Answer A is incorrect). Although vasopressin could be added to norepinephrine in septic shock, it is not recommended as the initial vasopressor of choice (Answer B is incorrect).
A 67-year-old woman presents to the ED with septic shock, presumably caused by a UTI. Her medical history is significant for hypertension, hyperlipidemia, and recurrent Escherichia coli UTIs. Pertinent vital signs include blood pressure 82/44 mm Hg, heart rate 106 beats/minute, respiratory rate 28 breaths/minute, and temperature 103.6ºF (39.8ºC). Pertinent laboratory values include WBC 24 × 103 cells/mm3, Na 148 mEq/L, K 3.8 mEq/L, Cl 110 mEq/L, HCO3 19 mEq/L, SCr 1.9 mg/dL, and lactate 4.1 mmol/L. On physical examination, the patient is confused and lethargic with a Glasgow Coma Scale score of 12, and she weighs 68 kg. She is given 2 L of lactated Ringer solution, cultures are sent, and broad-spectrum antibiotics are being administered.
Question 7
In which scenario would hydrocortisone most be indicated for this patient?
A Result from a corticotropin stimulation test yielding a cortisol change of 7 mcg/mL. B Norepinephrine 0.4 mcg/kg/minute, vasopressin 0.03 units/minute, and blood pressure 88/42 mm Hg. C Norepinephrine 0.05 mcg/kg/minute and blood pressure 94/52 mm Hg. D Hydrocortisone not indicated in septic shock.
B - norepinephrine 0.4 mcg/kg/minute, vasopressin 0.03 units/minute, and blood pressure 88/42 mm Hg, is correct. The patient remains hypotensive despite increased doses of vasopressor therapy (Answer D is incorrect). The Surviving Sepsis Campaign guidelines recommend against using hydrocortisone in patients for whom hemodynamic stability was restored after adequate fluid resuscitation and vasopressor therapy (Answer C is incorrect). The corticotropin stimulation test is not recommended to identify patients who should receive hydrocortisone (Answer A is incorrect).
R.M. is a 68-year-old woman (height 65 inches, weight 65 kg) with newly diagnosed atrial fibrillation. Her medical history includes dyslipidemia, hypertension, peptic ulcer disease 2 years ago, chronic kidney disease, and hypothyroidism. Her current medications include metoprolol tartrate 50 mg twice daily, atorvastatin 20 mg once daily, chlorthalidone 25 mg once daily, dronedarone 400 mg twice daily, omeprazole 20 mg once daily, and levothyroxine 0.25 mg once daily. Her laboratory values include K 4.2 mEq/L, blood glucose 98 mg/dL, and calculated CrCl 40 mL/minute/1.73 m2. Her vital signs today include heart rate 78 beats/minute and blood pressure 128/78 mm Hg.
Question 8
Which is the most appropriate regimen for reducing R.M.’s risk of stroke?
A Dabigatran 75 mg twice daily. B Rivaroxaban 20 mg once daily. C Apixaban 2.5 mg twice daily. D Edoxaban 60 mg once daily.
A - Patients with a CrCl of less than 50 mL/minute/1.73 m2 should have their rivaroxaban dose decreased from 20 mg daily to 15 mg daily, making Answer B incorrect. The apixaban dose is only decreased to two of three criteria (SCr greater than 1.5 mg/dL, age 80 or older, or weight 60 kg or less). Although no SCr value is given, neither the patient’s age nor her weight would qualify for a dose reduction of apixaban, making Answer C incorrect. Answer D is incorrect because the edoxaban dose should be reduced with moderate renal insufficiency or with the use of dronedarone. The 75-mg twice-daily dabigatran dose is used with a CrCl of 15–30 mL/minute/1.73 m2 and with a CrCl of 30–50 mL/minute/1.73 m2 with the use with dronedarone or ketoconazole. Because this patient has moderate renal insufficiency and is taking dronedarone, Answer A is correct.
R.M. is a 68-year-old woman (height 65 inches, weight 65 kg) with newly diagnosed atrial fibrillation. Her medical history includes dyslipidemia, hypertension, peptic ulcer disease 2 years ago, chronic kidney disease, and hypothyroidism. Her current medications include metoprolol tartrate 50 mg twice daily, atorvastatin 20 mg once daily, chlorthalidone 25 mg once daily, dronedarone 400 mg twice daily, omeprazole 20 mg once daily, and levothyroxine 0.25 mg once daily. Her laboratory values include K 4.2 mEq/L, blood glucose 98 mg/dL, and calculated CrCl 40 mL/minute/1.73 m2. Her vital signs today include heart rate 78 beats/minute and blood pressure 128/78 mm Hg.
Question 9
R.M. was initiated on apixaban, and 6 months later, she is in a motor vehicle accident with major life-threatening bleeding. Which is the most appropriate treatment for her bleeding episode?
A Idarucizumab 5 g intravenously. B 8 units of fresh frozen plasma with vitamin K 10 mg intravenously. C Andexanet 400-mg intravenous bolus followed by 4 mg/minute for 2 hours. D 4-factor prothrombin complex concentrate 100 units/kg.
C - Andexanet alfa is the reversal agent for apixaban
A new anticoagulant was evaluated in a clinical trial to determine whether it provided efficacy similar to warfarin for reducing the risk of stroke in patients with atrial fibrillation. The study was designed to be a randomized, double-blind, noninferiority trial set to enroll 12,000 patients. Noninferiority was considered proven if there was no more than a 15% excess of events in the new anticoagulant arm of the study. The final results of the trial showed a hazard ratio of 0.96 and a 95% confidence interval of 0.80–1.18. Which is the most accurate interpretation of this clinical trial?
A New anticoagulant was noninferior to warfarin for reducing stroke. B New anticoagulant was equivalent to warfarin for reducing stroke. C New anticoagulant was not noninferior to warfarin for reducing stroke. D New anticoagulant showed a 4% absolute reduction in stroke.
C - A noninferiority trial is designed to determine whether a treatment is not clinically inferior to an existing therapy. This trial allowed for up to a 15% excess of events compared with warfarin therapy, which meant the upper bounds of the 95% confidence interval could not exceed 1.15. In this study, the upper end of the confidence interval was 1.18, which exceeded the study’s preset margin of noninferiority. Even though the hazard ratio was less than 1, the new anticoagulant did not prove to be noninferior to warfarin therapy, making Answer C correct and Answer A incorrect. Answer B is incorrect because a hazard ratio crossing 1 in a noninferiority trial does not prove equivalence of the two agents studied. The absolute end point event rates are not provided; therefore, the absolute difference cannot be calculated, making Answer D incorrect.
A 42-year-old man presents to the physician with shortness of breath and difficulty breathing over the past 6 months. His medical history is significant for hypertension. He denies any history of cigarette smoking or significant exposure. His family history is significant for chronic obstructive pulmonary disease (COPD) in his father (diagnosed at age 40) and severe asthma in a sister. He describes his symptoms as being worse at night than during the day, and they seem to worsen when he has spent time outside. He has tried his sister’s albuterol inhaler without improvement in symptoms. He denies any acutely worsening symptoms that have led to medical treatment other than “being tired of having a hard time breathing.” Spirometry testing reveals a forced expiratory volume in 1 second (FEV1) of 75% of predicted and an FEV1/forced vital capacity of 68%, with no response to a bronchodilator during testing. Which initial treatment approach is most appropriate for this patient?
A Inhaled corticosteroid–containing treatment is essential because the patient presents with clinical features of asthma. B Inhaled corticosteroid–containing treatment is essential in addition to a long-acting bronchodilator according to the GOLD recommendations because the patient presents with clinical features of both asthma and COPD. C The patient presents with clinical features of acute bronchitis and should not be treated at this time. D The patient presents with clinical features of COPD and should be treated according to the GOLD recommendations.
B - The patient presents with clinical features of both asthma and COPD. The patient’s age, family history of COPD, and spirometry testing indicate COPD. However, his lack of exposure to risk factors, family history of asthma, and worsening of symptoms at nighttime and after exposure to allergens indicate asthma. Given that his symptoms are consistent with both asthma and COPD, the patient should be initiated on inhaled corticosteroid–containing treatment in addition to COPD treatments according to the GOLD recommendations, making Answer B correct and Answers A, C, and D incorrect
M.C. is a 42-year-old man who stepped on a nail a month ago. The nail penetrated his shoe and went deep into his left foot. He cleaned the puncture area well and applied antibiotic cream to the area for about a week. He now presents with pain and swelling in the left foot. An MRI of the foot reveals osteomyelitis with bone erosion. A bone biopsy with tissue culture is planned but not completed at this time. Which is the best empiric antibiotic regimen for M.C.?
A Nafcillin for 2 weeks, followed by dicloxacillin to complete a 4-week course. B Cefepime for 6 weeks. C Vancomycin plus metronidazole for 6 weeks. D Levofloxacin for 2 weeks.
B - Although nafcillin is commonly used first line for osteomyelitis, Pseudomonas aeruginosa is often involved in foot puncture osteomyelitis. Therefore, a broader-spectrum antibiotic is required. In addition, oral antibiotics should only be used if certain characteristics are present (Answer A is incorrect). Cefepime is the best option for this patient. Cefepime has good activity against Pseudomonas and gram-positive organisms, both of which are possible causative organisms in this case. In addition, the length of therapy is appropriate (4–6 weeks) (Answer B is correct). Vancomycin and metronidazole would not have activity against Pseudomonas (Answer C is incorrect). Levofloxacin might not achieve adequate concentrations at the infection site to kill gram-positive organisms, and the length of therapy for this option is too short (Answer D is incorrect).
In a study reporting the occurrence of patient symptoms, participants were asked to rate the severity of their symptoms on a scale from 0 to 5 (0 = no symptoms, 5 = unbearable symptoms). Which measure of central tendency is best to describe patients’ responses to this question?
A Mean. B Interquartile range. C Median. D Standard deviation (SD).
C - Ordered or Likert-type scales typically are not continuous. For example, the interval between 1 and 2 is not necessarily the same as that between 2 and 3. Although there is some controversy about a true Likert scale, using means and SDs to describe the central tendency and variability of these types of data are among the most common errors made in the literature (Answer A is incorrect). Answers B and D are incorrect because they are measures of spread or variability, not point estimates of the central tendency. Answer C, median, provides the most appropriate description of the central tendency of these types of data.
A 61-year-old woman with chronic obstructive pulmonary disease (COPD) was recently admitted to the hospital because of a moderate exacerbation. This is the patient’s third exacerbation in the past year. She is a current smoker (42 pack-years) and is not interested in quitting within the next 6 months. She verbalizes appropriate doses and frequencies of all inhalers, including umeclidinium/vilanterol once daily and albuterol metered dose inhaler (MDI) as needed. A medication adherence report shows proportion of days covered (PDC) as 99% and 98% for umeclidinium/vilanterol and albuterol MDI, respectively. Which educational strategy is most appropriate to help the patient improve COPD control and prevent exacerbations?
A Ask the patient to demonstrate inhaler administration technique. B Provide education on nicotine replacement therapy to assist with a quit attempt. C Recommend COPD pharmacotherapy adjustments and assess patient understanding using the teach-back method. D Reinforce the dosing and frequency of current inhalers.
A - Studies have shown a significant relationship between poor inhaler use and worsening symptom control in patients with COPD. Regular education on inhaler technique is critically important and should always be assessed before assuming current therapy is inadequate (Answer A is correct; Answer C is incorrect). Although the patient would benefit from tobacco cessation, she is currently in the precontemplation stage. Given the patient’s current stage of change, recommending nicotine replacement therapy would not be appropriate at this time, making Answer B incorrect. The patient should be provided information, with permission, about the health consequences of smoking with the goal of moving her to the contemplation stage. Although reinforcement of dosing and frequency of inhaler use would be appropriate, the patient’s self-report of administering doses as prescribed in addition to high PDC does not support this as the main concern at this time (Answer D is incorrect).
A.R. is a 62-year-old white man (height 68 inches, weight 88 kg) with a history of hypertension, a non–ST-segment elevation myocardial infarction 8 months ago with two drug-eluting stents placed, chronic kidney disease (baseline SCr 2.7 mg/dL), New York Heart Association (NYHA) class II heart failure (HF) with a left ventricular ejection fraction of 30%, and chronic lower back pain. His current medications include enalapril 10 mg twice daily, furosemide 40 mg daily, atenolol 50 mg daily, aspirin 81 mg daily, clopidogrel 75 mg daily, atorvastatin 40 mg daily, and naproxen 550 mg daily. He is following up in the clinic today for management of his HF regimen. He has no new concerns. On examination, his breath sounds are clear, he has no evidence of edema, his blood pressure is 128/76 mm Hg, and his heart rate is 78 beats/minute. His laboratory values include Na 141 mEq/L, K 4.2 mEq/L, BUN 44 mg/dL, and SCr 2.7 mg/dL.
Question 15
Which additional medication changes would be best to make to A.R.’s regimen?
A Discontinue clopidogrel. B Change naproxen to acetaminophen. C Change atorvastatin to simvastatin 40 mg daily. D Add losartan 50 mg daily.
B - A.R. recently had a myocardial infarction (MI) and received drug-eluting stents, so both clopidogrel (Answer A) and a high-intensity statin (Answer C) should be continued. Although losartan is a reasonable alternative to angiotensin-converting enzyme (ACE) inhibitor therapy in heart failure with reduced ejection fraction (HFrEF), angiotensin receptor blockers generally should not be used in combination with ACE inhibitors because of the increased risk of kidney injury and hyperkalemia (Answer D is incorrect). Naproxen is an NSAID, which can promote sodium and water retention, blunt diuretic response, and increase the risk of cardiovascular events and mortality in a patient with a history of HF and MI. Therefore, naproxen should be discontinued and changed to acetaminophen, which is considered safe in HF (Answer B is correct).
A.R. is a 62-year-old white man (height 68 inches, weight 88 kg) with a history of hypertension, a non–ST-segment elevation myocardial infarction 8 months ago with two drug-eluting stents placed, chronic kidney disease (baseline SCr 2.7 mg/dL), New York Heart Association (NYHA) class II heart failure (HF) with a left ventricular ejection fraction of 30%, and chronic lower back pain. His current medications include enalapril 10 mg twice daily, furosemide 40 mg daily, atenolol 50 mg daily, aspirin 81 mg daily, clopidogrel 75 mg daily, atorvastatin 40 mg daily, and naproxen 550 mg daily. He is following up in the clinic today for management of his HF regimen. He has no new concerns. On examination, his breath sounds are clear, he has no evidence of edema, his blood pressure is 128/76 mm Hg, and his heart rate is 78 beats/minute. His laboratory values include Na 141 mEq/L, K 4.2 mEq/L, BUN 44 mg/dL, and SCr 2.7 mg/dL.
Question 16
Which is the best recommendation for management of A.R.’s HF regimen?
A Increase enalapril to 20 mg twice daily. B Add hydralazine/isosorbide dinitrate 1 tablet three times daily. C Add spironolactone 12.5 mg daily. D Change atenolol to metoprolol succinate.
D - Answer A is incorrect; this patient is already taking the target dose of enalapril and has achieved a goal blood pressure, so further increases are unnecessary. Answer B is incorrect because hydralazine/isosorbide dinitrate is recommended in patients self-described as African American who are already receiving optimal doses of angiotensin-converting enzyme (ACE) inhibitors and ß-blockers. Although spironolactone is a reasonable add-on therapy in patients with NYHA class II–IV heart failure with reduced ejection fraction (HFrEF) receiving ACE inhibitors and ß-blockers, this patient’s SCr is greater than 2.5 mg/dL, so aldosterone receptor antagonists should be avoided because of the risk of hyperkalemia (Answer C is incorrect). ß-Blockers are recommended in all patients with HFrEF in addition to ACE inhibitors unless contraindicated. However, only bisoprolol, carvedilol, and metoprolol succinate have been shown to improve mortality in HFrEF, making Answer D correct.
K.P. is a 39-year-old woman with constipation-predominant irritable bowel syndrome (IBS-C). She also has a history of heartburn, for which she is prescribed famotidine 20 mg twice daily, but has difficulty remembering the evening dose. K.P. recently had a reduction in stool frequency despite using psyllium 1 scoop daily and pegylated interferon 17 g daily. She reports that her last bowel movement was 3 days ago. She also has concerns of abdominal discomfort and bloating. Which is the best recommendation for K.P. at this time?
A Increase the psyllium dose to 1 scoop twice daily. B Add plecanatide 3 mg daily. C Add tegaserod 6 mg twice daily. D Add lubiprostone 8 mcg twice daily.
B - This patient needs additional therapy for IBS-C, for which secretagogues such as lubiprostone, linaclotide, and plecanatide are effective. She reports difficulty remembering her second daily dose of famotidine; thus, lubiprostone, which is administered twice daily, might also be difficult for her to take consistently (Answer D is incorrect). A medication dosed once per day such as plecanatide would be a better option for her (Answer B is correct). Increasing the psyllium dose could exacerbate her abdominal discomfort and bloating (Answer A is incorrect). Tegaserod is only available on a limited basis for emergency use because of its risk of cardiovascular adverse effects (Answer C is incorrect).
A 28-year-old man with schizophrenia has been stable on clozapine for 1 year. He is brought to the ED after a seizure. He is somnolent and unable to walk or stand. He has a smoking history of 1 pack/day for 8 years. He started varenicline 2 weeks ago to help him stop smoking. His last cigarette was 1 week ago. He drinks 1–2 cups of coffee daily and had 1 beer last night. Which is the most likely cause of this patient’s current clinical picture?
A Caffeine intake. B Combining alcohol with varenicline. C Combining clozapine and varenicline. D Smoking cessation
D - Clozapine is metabolized by CYP1A2 and Smoking is an inducer, when the patient stopped smoking his clozapine concentrations increased.
A 42-year-old woman wants to stop smoking. She has a 22 pack-year history and currently smokes 1 pack/day. She has her first cigarette immediately upon awakening. Her medical history includes anorexia nervosa, schizophrenia, nasal polyps, and asthma. Her current medications include risperidone long-acting injectable every 2 weeks, budesonide 160 mg/formoterol fumarate 4.5 mg 2 puffs twice daily, and albuterol HFA every 4–6 hours as needed (using twice weekly). This is her first quit attempt, and she is interested in a highly effective product. Which treatment would be best to recommend for her?
A Bupropion. B Nicotine gum. C Nicotine nasal spray. D Varenicline.
D - Varenicline is the best option for this woman (Answer D is correct). The latest data analyses suggest that varenicline is not as likely to cause neuropsychiatric symptoms as once thought, and a review of existing trials shows that patients with schizophrenia are not more likely to develop neuropsychiatric symptoms. The advantages of abstinence from smoking are believed to outweigh any risk of developing psychiatric symptoms. Bupropion is contraindicated in patients with eating disorders, and this patient has a history of anorexia nervosa (Answer A is incorrect). Nicotine gum monotherapy is less likely to be effective for smoking cessation than if it were combined with nicotine patches or other modalities (Answer B is incorrect). Nicotine nasal spray should be avoided in patients with nasal polyps (Answer C is incorrect).
S.K. is an 18-year-old woman brought to your ED for a seizure that has lasted 10 minutes. She has no history of epilepsy and does not take any medications. Which is best for treatment of S.K.’s seizure?
A No treatment. B Levetiracetam. C Lorazepam. D Valproate.
C - Status epilepticus is defined as a prolonged seizure lasting more than 5 minutes. Because this seizure has lasted for 10 minutes, treatment is required, making Answer A incorrect. Levetiracetam (Answer B) can be used if prior treatments fail, but not in the initial treatment of status epilepticus. Similarly, valproate (Answer D) can be used in refractory status epilepticus. In addition, valproate is not ideal for a woman who might be pregnant. Lorazepam (Answer C) is used as initial treatment in status epilepticus to control seizures.
The Centers for Medicare & Medicaid Services (CMS) implements quality initiatives to ensure quality health care for patients receiving Medicare. This has produced the National Hospital Inpatient Quality Measures, which are aligned with The Joint Commission. Which best depicts the disease state that is part of these quality measures for 30-day risk-standardized mortality?
A Hip replacement surgery. B Hypertensive crises. C Stroke. D Atrial fibrillation.
C - Acute Myocardial Infarction Heart Failure Pneumonia Chronic Obstructive Pulmonary Disease (COPD) Coronary Artery Bypass Graft (CABG) Stroke
are the measures. Hip/knee and readmission is the 30 day readmission measure
Which of the following infections is a notifiable disease and should be reported to the Centers for Disease Control and Prevention when diagnosed?
A ESBL producing Klebsiella pneumoniae. B Vancomycin resistant Enterococcus faecium. C Listeria monocytogenes. D Clostridioides difficile.
C - CDC notifiable diseases are considered of significant public health importance and help the CDC effectively follow trends in infections or disease outbreaks. Many of the more common bacterial infections are not considered “notifiable.” Although ESBL producing organisms are concerning from an antibiotic resistance perspective, they are not notifiable (Answer A is incorrect). Vancomycin intermediate or resistant Staphylococcus aureus is notifiable but not VRE (Answer B is incorrect). C. difficile infections are important to track from a local health system perspective, but when diagnosed the CDC does not need to be notified (Answer D is incorrect). Since L. monocytogenes infections can originate from contaminated foods, is it important for the CDC to track these infections to potentially recall any contaminated products and prevent a widespread outbreak.
A 48-year-old woman is brought to your institution by ambulance after being found unconscious at home. She was intubated in the field, and laboratory values obtained in the ambulance show a sodium concentration of 183 mg/dL. She seized en route and received 2 mg of intravenous lorazepam. When the family arrives, you learn that she has a history of diabetes insipidus and was recently changed from intranasal to oral desmopressin. Which is the best initial step in this patient’s treatment?
A Sodium chloride 0.9%. B Sodium chloride 0.45%. C Sodium chloride 0.9% plus desmopressin 2 mcg intravenously. D Sodium chloride 0.45% plus desmopressin 0.1 mg orally.
C - Given this patient’s history of diabetes insipidus, she needs desmopressin as part of her treatment of hypernatremia. Because Answers A and B provide only fluid and not desmopressin, they are both incorrect. The patient is intubated and was found unconscious, so she should receive desmopressin intravenously rather than orally. In addition, the sodium content of sodium chloride 0.45% would decrease the patient’s sodium too quickly (Answer D). Sodium chloride 0.9% and intravenous desmopressin is the best choice for this patient (Answer C)
A patient with chronic kidney disease category G3a is scheduled for a cardiac catheterization for evaluation of possible coronary artery disease. Which treatment is best for preventing contrast-associated nephropathy?
A Acetylcysteine orally. B 0.9% sodium chloride intravenously. C Furosemide intravenously. D Sodium bicarbonate intravenously.
B - The best treatment for preventing contrast-associated nephropathy is volume expansion with intravenous saline (Answer B is correct). Although diuretics may increase urinary output, there is no evidence that they decrease the risk of nephropathy, and they may worsen nephropathy if volume status is worsened (Answer C is incorrect). Although early data analyses suggested a benefit from intravenous sodium bicarbonate, more recent information has shown no benefit (Answer D is incorrect). Although acetylcysteine has been used because it is safe and inexpensive, current data analyses suggest this agent does not prevent contrast-associated acute kidney injury (Answer A is incorrect).
B.L. is a 68-year-old woman (height 64 inches, weight 58 kg) with a medical history of hypertension and heart failure (left ventricular ejection fraction 35%). Her current medications include lisinopril 20 mg daily and metoprolol succinate 50 mg daily. She presents to the clinic today with concerns about increased shortness of breath when walking and a “fluttering in my chest.” Her blood pressure today is 134/82 mm Hg and heart rate is 98 beats/minute. The patient has no other concerns. On examination, her breath sounds are clear, and she has no pitting edema. An ECG reveals atrial fibrillation (AF). Her laboratory results are all within normal limits, including SCr 1.2 mg/dL and BNP 84 pg/mL. Which is the best pharmacologic approach for managing B.L.’s symptoms today?
A Increase metoprolol succinate to 100 mg daily. B Add diltiazem CD 240 mg daily. C Add dronedarone 400 mg twice daily. D No changes are needed; B.L.’s heart rate is at goal.
A - B.L. has been given a diagnosis of AF and is symptomatic, so her heart rate goal is less than 80 beats/minute. A heart rate goal of less than 110 beats/minute would be reasonable only if B.L. were asymptomatic, making Answer D incorrect. An initial rate control strategy is preferred. Furthermore, dronedarone would be an inappropriate antiarrhythmic agent for this patient, given her history of heart failure with reduced ejection fraction (HFrEF) (Answer C). Both metoprolol succinate and diltiazem could improve the patient’s heart rate, but diltiazem is not recommended in patients with HFrEF because of its negative inotropic properties when used in HFrEF (Answer B). Increasing the dose of metoprolol succinate should help improve B.L.’s heart rate, making Answer A correct. In addition, metoprolol succinate is not yet at the target dose for her HFrEF and should be titrated to maximally tolerated doses or a goal of 200 mg daily.
A 25-year-old woman with a history of recurrent major depressive disorder has been admitted to the inpatient psychiatric unit after being treated for an overdose of acetaminophen and alprazolam (not her prescription). She has obesity and insomnia. She had anorgasmia while taking fluoxetine and worsened insomnia while taking venlafaxine. Which medication would be most appropriate to initiate in this patient?
A Bupropion. B Nortriptyline. C Paroxetine. D Vilazodone.
D - The antidepressant prescribed for this patient should be relatively safe in overdose. Tricyclic antidepressants such as nortriptyline can be fatal if too much is taken. Adverse effects include torsades de pointes and seizures, and overdoses are often fatal (Answer B is incorrect). Paroxetine is associated with a high rate of sexual dysfunction and can cause weight gain (Answer C is incorrect). Although bupropion does not cause sexual dysfunction, it can cause seizures in a dose-dependent fashion. This makes bupropion a poor choice in a patient with suicidal ideation, and it should be avoided in patients with seizure disorder. Bupropion is also activating and could worsen the patient’s insomnia (Answer A is incorrect). Vilazodone has a lower incidence of sexual dysfunction and is relatively safe in overdose, making it the best choice (Answer D is correct).
Enteral nutrition is being initiated in a 72-year-old man (height 69 inches, weight 72 kg) in your ICU after a stroke. He was well nourished before admission, and his laboratory values are within normal limits. Which would be the most appropriate goals in this patient?
A 1650 kcal plus 72 g of protein. B 1725 kcal plus 144 g of protein. C 2000 kcal plus 101 g of protein. D 2200 kcal plus 86 g of protein.
C - The patient requires 25–30 kcal/kg/day and 1.3–1.5 g/kg/day of protein
M.H., an 83-year-old woman, presents to her ambulatory care clinic accompanied by her daughter, who reports she no longer feels comfortable leaving her mother alone because of her mother’s “increasing forgetfulness.” The patient reports increasing pain at her knee joints, contributing to difficulty walking. The patient’s medical history is significant for type 2 diabetes, hypertension, coronary artery disease, congestive heart failure, and osteoarthritis. She takes the following medications: aspirin 81 mg daily, atorvastatin 20 mg once daily, lisinopril 20 mg daily, furosemide 20 mg daily, potassium chloride 20 mEq daily, carvedilol 12.5 mg twice daily, metformin 1000 mg twice daily, and glipizide extended release 5 mg daily. Her Mini-Mental State Examination (MMSE) score is 20/30, and her Geriatric Depression Scale score is 3/15. Blood tests obtained last week showed a normal basic metabolic panel, except for a fasting plasma glucose of 65 mg/dL. Her A1C is 7.6%. A urinalysis is negative. No nutritional deficiencies are noted. The patient’s blood pressure is 130/80 mm Hg and heart rate is 60 beats/minute. She is given a diagnosis of Alzheimer disease (AD), and initiation of a cholinesterase inhibitor is considered.
Question 29
Although initiating a cholinesterase inhibitor for M.H. might improve her cognition, which adverse effect would it most likely cause?
A Vivid dreams. B Depression. C Hallucinations. D Diarrhea.
D - Cholinesterase inhibitors prevent the breakdown of acetylcholine, resulting in increased cholinergic activity. Cholinesterase inhibitors in older adults with dementia can cause or worsen diarrhea because of increased cholinergic activity (Answer D is correct). Vivid dreams, depression, and hallucinations are other possible adverse effects to consider monitoring for when initiating cholinesterase inhibitor therapy but are less likely to occur than GI adverse effects (Answers A, B, and C are incorrect).
M.H., an 83-year-old woman, presents to her ambulatory care clinic accompanied by her daughter, who reports she no longer feels comfortable leaving her mother alone because of her mother’s “increasing forgetfulness.” The patient reports increasing pain at her knee joints, contributing to difficulty walking. The patient’s medical history is significant for type 2 diabetes, hypertension, coronary artery disease, congestive heart failure, and osteoarthritis. She takes the following medications: aspirin 81 mg daily, atorvastatin 20 mg once daily, lisinopril 20 mg daily, furosemide 20 mg daily, potassium chloride 20 mEq daily, carvedilol 12.5 mg twice daily, metformin 1000 mg twice daily, and glipizide extended release 5 mg daily. Her Mini-Mental State Examination (MMSE) score is 20/30, and her Geriatric Depression Scale score is 3/15. Blood tests obtained last week showed a normal basic metabolic panel, except for a fasting plasma glucose of 65 mg/dL. Her A1C is 7.6%. A urinalysis is negative. No nutritional deficiencies are noted. The patient’s blood pressure is 130/80 mm Hg and heart rate is 60 beats/minute. She is given a diagnosis of Alzheimer disease (AD), and initiation of a cholinesterase inhibitor is considered.
Question 30
Which patient parameter most justifies initiating a cholinesterase inhibitor for M.H. right now?
A MMSE score 20/30. B Geriatric Depression Scale score 3/15. C Increasing forgetfulness reported by the patient’s daughter. D A1C greater than 6.5%.
A - For mild to moderate AD, a cholinesterase inhibitor is initially the most appropriate option for slowing cognitive decline; this patient’s MMSE score of 20/30 and corresponding diagnosis of mild dementia warrant initiating cholinesterase inhibitor therapy (Answer A is correct). The Geriatric Depression Scale score does not correspond with justification of use of cholinesterase inhibitor therapy (Answer B is incorrect). Further exploration of the daughter’s reports of her mother’s increasing forgetfulness would help classify the severity of her mother’s cognitive impairment and AD diagnosis but, alone, would not be adequate to justify the need for a cholinesterase inhibitor (Answer C is incorrect). Finally, A1C is related to the patient’s diagnosis of diabetes and is not relevant to the question regarding initiation of a cholinesterase inhibitor (Answer D is incorrect).
If MMSE is lower than 24 it means decline in cognitive function.
M.H., an 83-year-old woman, presents to her ambulatory care clinic accompanied by her daughter, who reports she no longer feels comfortable leaving her mother alone because of her mother’s “increasing forgetfulness.” The patient reports increasing pain at her knee joints, contributing to difficulty walking. The patient’s medical history is significant for type 2 diabetes, hypertension, coronary artery disease, congestive heart failure, and osteoarthritis. She takes the following medications: aspirin 81 mg daily, atorvastatin 20 mg once daily, lisinopril 20 mg daily, furosemide 20 mg daily, potassium chloride 20 mEq daily, carvedilol 12.5 mg twice daily, metformin 1000 mg twice daily, and glipizide extended release 5 mg daily. Her Mini-Mental State Examination (MMSE) score is 20/30, and her Geriatric Depression Scale score is 3/15. Blood tests obtained last week showed a normal basic metabolic panel, except for a fasting plasma glucose of 65 mg/dL. Her A1C is 7.6%. A urinalysis is negative. No nutritional deficiencies are noted. The patient’s blood pressure is 130/80 mm Hg and heart rate is 60 beats/minute. She is given a diagnosis of Alzheimer disease (AD), and initiation of a cholinesterase inhibitor is considered.
Question 31
Which would be the most appropriate treatment for M.H.’s OA knee pain?
A Ibuprofen 200 mg four times daily. B Diclofenac gel 1% applied four times daily to the affected joints. C Tramadol 50 mg three times daily as needed for pain. D Acetaminophen 650 mg three times daily.
B - An initial trial of a topical NSAID is reasonable for patients with OA pain because the efficacy of topical NSAIDs is similar to systemic NSAIDs without most of the adverse effects (Answer B is correct). Ibuprofen would be an alternative when topical NSAIDs have failed (Answer A is incorrect). Given the patient’s history of coronary artery disease and heart failure, as well as the fact that she takes aspirin daily, using ibuprofen would also be discouraged. As-needed tramadol should be used cautiously in older patients and only when NSAIDs are contraindicated or ineffective and the pain is severe (Answer C is incorrect). Acetaminophen at doses of less than 3 g/day is a potential alternative, but many studies have shown minimal efficacy, and long-term use requires monitoring for hepatotoxicity (Answer D is incorrect).
A patient with type 2 diabetes was recently initiated on a basal-bolus insulin regimen. The current basal dose is insulin glargine 20 units once daily. The current bolus dose is insulin aspart 7 units before breakfast, lunch, and dinner. When implementing a correctional insulin dose strategy to minimize hyperglycemic excursions, which is the best estimate, in milligrams per deciliter, of how much 1 unit of insulin aspart would reduce this patient’s plasma glucose concentration?
A 11. B 22. C 31. D 44.
D - To estimate how much in milligrams per deciliter 1 unit of a rapid-acting insulin would decrease an individual patient’s glucose concentration, the rule of 1800 applies. This patient’s total daily insulin needs, adding the current basal and bolus insulin regimen, calculates as 41. Dividing the total daily insulin needs into 1800 is 44. Hence, 1 unit of insulin aspart would be expected to decrease a glucose concentration by 44 mg/dL (Answer D is correct). Answers A, B, and C would decrease glucose concentrations by 163, 82, and 58 mg/dL, respectively.
A 78-year-old man with chronic obstructive pulmonary disease (COPD) has been having worsening symptoms of cough, sputum production, and shortness of breath over the past 2 months. He has had two documented exacerbations in the past 9 months, with the most recent requiring hospitalization 1 month ago. He currently has moderate symptoms with most activities. His CAT (COPD Assessment Test) score is 14 today. His most recent spirometry reveals a forced expiratory volume in 1 second (FEV1) of 48% of predicted. At last check, his eosinophil count was 175 cells/microliter. His current treatment for COPD is indacaterol/glycopyrrolate 27.5/15.6 mcg 1 capsule inhaled twice daily and albuterol 100 mcg 1 puff four times daily as needed for shortness of breath/wheezing.
Question 33
Two months ago, the patient began the treatment regimen recommended, and his symptoms have improved. Today, he presents to the ED with concerns of increased trouble breathing, cough, and sputum production with a purulent color. On examination, his respiratory rate is 22 breaths/minute, SaO2 is 89%, and CrCl is 45 mL/minute/1.73 m2. He is alert and oriented × 3. After receiving supplemental oxygen and albuterol 2.5 mg by nebulizer, his shortness of breath is improving. Which is the most appropriate regimen to initiate today?
A Prednisone 40 mg orally once daily for 5 days. B Prednisone 40 mg orally once daily for 14 days. C Prednisone 40 mg orally once daily for 5 days and antibiotic guided by sputum culture results. D Prednisone 40 mg orally once daily for 14 days and antibiotic guided by sputum culture results.
C - The patient is currently experiencing a severe COPD exacerbation, given his respiratory rate and FEV1. He is being treated in the ED appropriately. Given the presence of the three cardinal symptoms (dyspnea, increased sputum volume and increased sputum purulence antibiotics are indicated (Answers A and B are incorrect). Systemic corticosteroids given for 5 days are noninferior to longer 14-day courses; therefore, a shorter duration should be used (Answers B and D are incorrect). Given that his last exacerbation requiring hospitalization was 3 months ago and he has severe COPD (FEV1 less than 50%), he is at risk of Pseudomonas. A sputum culture should be obtained to determine the presence of gram-negative bacteria or resistant pathogens to guide antibiotic treatment decisions (Answer C is correct).
A 78-year-old man with chronic obstructive pulmonary disease (COPD) has been having worsening symptoms of cough, sputum production, and shortness of breath over the past 2 months. He has had two documented exacerbations in the past 9 months, with the most recent requiring hospitalization 1 month ago. He currently has moderate symptoms with most activities. His CAT (COPD Assessment Test) score is 14 today. His most recent spirometry reveals a forced expiratory volume in 1 second (FEV1) of 48% of predicted. At last check, his eosinophil count was 175 cells/microliter. His current treatment for COPD is indacaterol/glycopyrrolate 27.5/15.6 mcg 1 capsule inhaled twice daily and albuterol 100 mcg 1 puff four times daily as needed for shortness of breath/wheezing.
Question 34
Which is the most appropriate maintenance medication adjustment to initiate today?
A Add roflumilast 500 mcg by mouth once daily. B Add tiotropium 2.5 mcg 2 puffs once daily. C Replace with fluticasone propionate/salmeterol 113/14 mcg 1 puff twice daily. D Replace with fluticasone furoate/umeclidinium/vilanterol 100/62.5/25 mcg 1 puff once daily.
Given the slow worsening of the patient’s symptoms over the past 2 months, treatment intensification should be recommended. According to the GOLD guidelines, therapy adjustments should be based on the predominant trait of dyspnea or exacerbations. Given this patient’s exacerbation history and recent hospitalization, the exacerbation pathway should be followed. Therapy adjustments should be made on the basis of current maintenance therapy. Because this patient is currently taking a combination long-acting muscarinic antagonist (LAMA)/long-acting ß2-agonist (LABA) and has an eosinophil count greater than 100 cells/microliter, adding an inhaled corticosteroid (ICS) is recommended (triple therapy with a LAMA/LABA/ICS), making Answer D correct. Answer A is incorrect because roflumilast should only be considered with an FEV1 of less than 50%, chronic bronchitis, and eosinophils less than 100 cells/microliter. Answer B is incorrect because this would be stepping down treatment, given that the patient is currently taking LAMA/LABA therapy. Answer C is incorrect because this option does not include a LAMA
A 3-year-old boy presented to your pediatric clinic yesterday with fever, decreased enteral intake, irritability, and tugging at his ears. He was given a diagnosis of acute otitis media (AOM) in both ears. A delayed antibiotic-prescribing strategy was decided on, and the patient was sent home with instructions for ibuprofen as needed. The boy’s father has brought him back to your clinic today because he seems more restless, and the fever is inadequately controlled with ibuprofen alone. The patient has a history of receiving amoxicillin for otitis media about 6 months ago. Which recommendation is best to treat this patient?
A Administer cefdinir oral solution at 14 mg/kg/day for 7 days. B Administer ceftriaxone 50 mg/kg intravenously for 3 days. C Administer amoxicillin oral solution at 90 mg/kg/day for 7 days. D Continue to watch and wait until the 48–72 hours have passed.
C - Delayed antibiotic prescribing is acceptable for children older than 6 months. For children older than 2 years, this approach is recommended only if the symptoms are mild and there is no otorrhea. This approach is also acceptable for bilateral AOM if the child is older than 2 years. For patients in whom delayed antibiotic prescribing is initiated, an antibiotic should be prescribed if symptoms worsen or if there is no improvement in 48–72 hours. In this case, the symptoms appear to be worse, so further treatment should not be delayed (Answer D is incorrect). Cefdinir and ceftriaxone may be considered in patients whose first-line treatment regimens have failed, but this patient has not yet been initiated on antibiotics (Answers A and B are incorrect). Cefdinir would have been reasonable, however, if the patient had a true allergy to penicillins. The American Academy of Pediatrics recommends high-dose amoxicillin (80–100 mg/kg/day) as first-line therapy for AOM (Answer C is correct).
A 55-year-old man with a history of hypertension, dyslipidemia, and coronary artery disease had a drug-eluting stent placed 6 months ago. He presents today with a non–ST-segment elevation myocardial infarction (NSTEMI) and a high thrombolysis in myocardial infarction (TIMI) risk score. Inpatient therapies include aspirin 81 mg orally daily, ticagrelor 180 mg (single oral dose), and an unfractionated heparin intravenous infusion. After 12 hours, there is resolution of ECG changes, but intermittent chest pain remains. Coronary angiography reveals worrisome three-vessel disease. He is scheduled for urgent coronary artery bypass grafting (CABG) surgery. Which is the best management strategy, given his recent antiplatelet administration?
A Discontinue ticagrelor; go ahead with surgery after 24 hours; continue aspirin. B Recommend that surgery be postponed for 7 days; discontinue ticagrelor and aspirin. C Recommend that surgery be postponed for 5 days; discontinue ticagrelor and continue aspirin. D Go ahead with surgery, and continue both ticagrelor 90 mg twice daily and aspirin
A - This patient has intermittent angina and presents with a high TIMI risk score, making him at high risk of a recurrent myocardial infarction at this time. The U.S. guidelines for treating patients undergoing urgent CABG surgery recommend discontinuing clopidogrel or ticagrelor for at least 24 hours in the setting of urgent CABG. For elective CABG, clopidogrel and ticagrelor should be discontinued for at least 5 days and prasugrel for at least 7 days before surgery. In addition, this patient had a drug-eluting stent placed 6 months ago. Guidelines recommend continuing a P2Y12 inhibitor for patients who had a stent placed less than 12 months ago; for patients at high risk of ischemic events, guidelines recommend continuing aspirin (Answer A is correct). Answers B and C because this is not an elective procedure. Answer B is also incorrect because aspirin should be continued. Answer D is incorrect because the P2Y12 inhibitor should be discontinued for at least 24 hours to minimize the risk of bleeding from CABG.
M.K. is a 59-year-old white woman with a medical history of hypertension, type 2 diabetes, and stable ischemic heart disease. Her current medications include aspirin 81 mg daily, metformin 500 mg twice daily, rosuvastatin 20 mg daily, amlodipine 5 mg daily, and metoprolol tartrate 25 mg twice daily. She presents to the clinic today with concerns about continued chest pain when she works in her garden or walks her dogs. The pain is predictable and subsides after a few minutes of rest. She has no other concerns. Her clinic vital signs include blood pressure 92/60 mm Hg and heart rate 59 beats/minute. Her laboratory results are all within normal limits, including SCr of 0.9 mg/dL. Which medication adjustments, if any, would be most appropriate to make at this visit for M.K.?
A Increase metoprolol tartrate to 50 mg twice daily. B Increase amlodipine to 10 mg once daily. C Add ranolazine 500 mg twice daily. D No medication changes are warranted.
C - Relieves chest pain without lowering BP or HR.
In analyzing several clinical pharmacokinetic software programs for your institution, you find two with most of the characteristics meeting your criteria. The main difference between the two programs is that only one can perform Bayesian pharmacokinetics. Which is the biggest advantage of analyzing your clinical data using Bayesian pharmacokinetics?
A When patient-specific data are limited, this method will place less emphasis on the population parameters. B With only a few serum concentrations, the calculated pharmacokinetic parameters will generally be more accurate. C The results of your analysis can be represented better graphically, including confidence intervals for each of the data points. D This method can effectively adjust for errors in documentation of the timing of drug concentration samples.
B - The biggest advantage of Bayesian pharmacokinetics is that, using population data, estimation of individual pharmacokinetic parameters is better with limited patient-specific data (Answer B is correct). This is accomplished by placing more emphasis on the population parameters when the patient-specific data are limited (Answer A is incorrect). Bayesian pharmacokinetics does not affect graphic output and cannot correct for inaccuracies in documentation of drug concentration sampling (Answers C and D are incorrect).
M.F. is a 36-year-old woman (height 66 inches, weight 56 kg) who presents to the ED with increased redness, swelling, and pain in her left leg. Duplex ultrasonography reveals a deep venous thrombosis (DVT). M.F. was hospitalized 2 weeks ago for pneumonia. She currently also has hypertension and smokes 1 pack/day of cigarettes. Her current medications include lisinopril 10 mg daily, hydrochlorothiazide 25 mg daily, and oral birth control pills. Her laboratory test results are all within normal limits. She plans to quit smoking and change her form of birth control.
Question 39
Which best depicts how long M.F. should receive anticoagulation therapy?
A 1 month. B 3 months. C 6 months. D Indefinitely.
B - All of the patient’s risk factors for venous thromboembolism (VTE) (recent hospitalization, smoking, oral birth control) are reversible. In patients with an identifiable and/or reversible cause of VTE, treatment recommendations are for 3 months of anticoagulant therapy, making Answer B correct. One month is only recommended for a superficial VTE, making Answer A incorrect. Answers C and D are incorrect because they would be considered in patients with idiopathic VTE.
M.F. is a 36-year-old woman (height 66 inches, weight 56 kg) who presents to the ED with increased redness, swelling, and pain in her left leg. Duplex ultrasonography reveals a deep venous thrombosis (DVT). M.F. was hospitalized 2 weeks ago for pneumonia. She currently also has hypertension and smokes 1 pack/day of cigarettes. Her current medications include lisinopril 10 mg daily, hydrochlorothiazide 25 mg daily, and oral birth control pills. Her laboratory test results are all within normal limits. She plans to quit smoking and change her form of birth control.
Question 40
Which is the most appropriate regimen for treating M.F.’s DVT?
A Enoxaparin 60 mg subcutaneously twice daily for 5 days, followed by edoxaban 60 mg once daily. B Unfractionated heparin 3400-unit bolus, followed by 670 units per hour with warfarin 5 mg daily. C Rivaroxaban 15 mg twice daily for 21 days, followed by 20 mg daily. D Apixaban 5 mg twice daily for 7 days, followed by 2.5 mg daily.
C - Although edoxaban does require at least 5 days of initial injectable anticoagulant therapy and enoxaparin 60 mg once daily is the correct dose, the edoxaban dose should be reduced to 30 mg once daily instead of 60 mg because the patient weighs less than 60 kg, making Answer A incorrect. Answer B is incorrect because venous thromboembolism (VTE) treatment with a direct oral anticoagulant is recommended over a traditional injectable followed by warfarin. Answer B is also incorrect because the dosing of unfractionated heparin is the correct dosing for an acute coronary syndrome, but not for VTE. Although apixaban can be used as pure oral therapy for VTE, the dosing is 10 mg twice daily for 7 days, followed by 5 mg twice daily. Therefore, Answer D is incorrect because of the incorrect dosing regimen. Answer C is correct for using rivaroxaban in the treatment of VTE.
A 72-year-old man is admitted to the medical ICU for post-cardiac arrest care with therapeutic hypothermia. His body temperature was maintained at 91.4ºF (33ºC) for 24 hours, and the health care team decides it is time to rewarm slowly at 0.5ºC every hour. Which is most important to consider during the rewarming phase?
A Frequent laboratory monitoring is necessary to guide potassium supplementation, given the risk of hypokalemia. B Vecuronium continuous infusion should be administered to prevent shivering. C Phenytoin should be administered for seizure prophylaxis. D Frequent blood glucose monitoring is necessary, given the risk of hypoglycemia.
D - Answer D, frequent blood glucose monitoring is necessary, given the risk of hypoglycemia, is correct. The cooling phase of therapeutic hypothermia may cause an intracellular shift of potassium, leading to a decreased serum potassium concentration on laboratory results. During the rewarming phase, an extracellular shift occurs, placing the patient at risk of hyperkalemia (Answer A is incorrect). Paralytics are not mandatory for shivering prevention during the rewarming phase and should be avoided, if possible (Answer B is incorrect). The rewarming phase of targeted temperature management alone does not place the patient at elevated risk of seizures, so prophylaxis with phenytoin is not required (Answer C is incorrect).
Differences in CYP inhibition may exist within the same class of therapeutic agents. Which class is best matched with the drug in that class that has the least CYP inhibition?
A Fluoroquinolones – ciprofloxacin. B Macrolides – clarithromycin. C Calcium channel blockers – diltiazem. D Histamine-2 blockers – famotidine.
D - Ciprofloxacin has significant CYP inhibition. The correct drug would be levofloxacin or moxifloxacin (Answer A is incorrect). Clarithromycin also has significant CYP inhibition. The correct drug would be azithromycin (Answer B is incorrect). Diltiazem has significant CYP inhibition. The correct drug would be any of the dihydropyridine calcium channel blockers (Answer C is incorrect). Famotidine does not inhibit CYP, whereas cimetidine does (Answer D is correct).`
Which step would most assist in clearly defining a clinical question using the PICO model?
A Compare databases to search. B Determine the preferred study design. C Evaluate the quality of evidence using the GRADE system. D Identify the intervention to be considered.
D - The PICO model is used to frame a clinical question in order to identify clinically relevant evidence in the literature. The “P” stands for patients or problems, in which the most important characteristics of the patient are described; “I” stands for the intervention to be considered (Answer D is correct); “C” stands for comparison or alternatives being considered compared with the intervention (Answer A is incorrect); and “O” represents the outcome of focus. A preferred study design may be identified according to the formulated clinical question using the PICO model, but the PICO model is not involved in defining the question itself, making Answer B incorrect. Although the GRADE system requires clearly defined clinical questions using PICO to evaluate quality of evidence, it is not included as part of the PICO model (Answer C is incorrect).
A health system is planning to expand its transitions of care program using the plan-do-study-act (PDSA) model. Which statement is most accurate when implementing the PDSA model for quality improvement?
A Process outcomes and measures should be defined during the “do” phase. B The PDSA cycle should be applied using a repetitive approach to allow for retesting and refinement. C The PDSA model should be implemented during large-scale quality improvement phases. D Unexpected observations should be documented during the “study” phase.
B - The PDSA model is a “test-and-learn-approach” to allow for refinement and continuous quality improvement through repetitive cycles, making Answer B correct. The PDSA cycle allows for controlled change experiments on a small scale before expanding to larger-scale projects (Answer C is incorrect). Process outcomes and measures should be defined during the “plan” phase, whereas active implementation in addition to documentation of the process, including barriers and unexpected observations, takes place during the “do” phase (Answers A and D are incorrect).
A 74-year-old woman with a history of hypertension and diabetes is admitted to the hospital with a 3-day history of nausea, vomiting, and diarrhea. Her baseline BUN and SCr are 15 mg/dL and 1.0 mg/dL, respectively. Her home medications include hydrochlorothiazide 12.5 mg by mouth daily, lisinopril 20 mg by mouth daily, and metformin 500 mg by mouth twice daily. She has had a diminished appetite and has been unable to drink fluids. Her blood pressure on admission is 102/64 mm Hg with a heart rate of 104 beats/minute (sitting) that decreases to 80/40 mm Hg on standing. Laboratory test results show serum sodium 136 mEq/L, serum potassium 4.2 mEq/L, BUN 48 mg/dL, SCr 2.2 mg/dL, and WBC 16.5 × 103 cells/mm3. A urinary catheter is placed, and a small amount of concentrated urine is obtained. Her urinary sodium is 12 mEq/L and urinary creatinine is 24.2 mg/dL. Which is the best assessment of this patient’s renal disease?
A Stage 1 prerenal acute kidney injury (AKI). B Stage 2 prerenal AKI. C Stage 1 intrinsic AKI. D Stage 2 intrinsic AKI.
B - This patient has evidence of AKI, given the increase in BUN and SCr within 24 hours. Her AKI is prerenal, given her history of fluid loss from vomiting and diarrhea with poor oral intake of fluids. Her low blood pressure with orthostatic hypotension is consistent with inadequate vascular volume. The BUN/SCr ratio (greater than 20:1) is consistent with prerenal azotemia, whereas it would be expected to be normal (10–15:1) in a patient with intrinsic AKI. The fractional excretion of sodium (FENa) of less than 1% is consistent with prerenal AKI. Using the following equation to calculate FENa [(urinary sodium/serum sodium)/(urinary creatinine/SCr)] × 100 = [(12/136)/(24.2/2.2)] × 100 yields 0.8%. This reflects increased sodium reabsorption by the kidneys. Intrinsic AKI from either acute interstitial nephritis or acute tubular necrosis would be characterized by sodium wasting, with a high urinary sodium concentration and high FENa. All of these suggest that Answers C and D are incorrect. This AKI is classified as stage 2 because the SCr has increased by 2.0–2.9 times baseline (2.2 times) (Answer B is correct), whereas it would need to be 1.5–1.9 times baseline to be classified as stage 1 (Answer A is incorrect).
A 74-year-old man presents to the clinic for a follow-up of benign prostatic hyperplasia and hypertension. His urinary symptoms are well controlled, blood pressure is 150/84 mm Hg, and heart rate is 60 beats/minute. On digital rectal examination, his prostate is smooth but enlarged. He is currently treated for hypertension with lisinopril 20 mg and atenolol 25 mg daily and, for benign prostatic hyperplasia, with tamsulosin 0.4 mg once daily with dinner. Which is currently the most appropriate course of action?
A Initiate finasteride 5 mg once daily. B Change tamsulosin to alfuzosin 10 mg once daily. C Increase atenolol to 50 mg daily. D Change tamsulosin to terazosin 5 mg daily.
D - In this patient with comorbid conditions of hypertension and benign prostatic hyperplasia, the choice of α-blocker is based on the adverse effect profiles. Changing tamsulosin to terazosin would improve the patient’s blood pressure control, but merely changing to an alternative selective α-blocker would not likely address the comorbid hypertension (Answer D is correct; Answer B is incorrect). Increasing the atenolol dose would not be appropriate, given the patient’s heart rate of 60 beats/minute (Answer C is incorrect). Initiating finasteride would not be indicated without first evaluating a prostate-specific antigen concentration (Answer A is incorrect).
A 54-year-old man with stage IIIa non–small cell lung cancer comes to the clinic today to start cycle 1 of chemotherapy with paclitaxel 45 mg/m2 and carboplatin dosed at AUC 2 given intravenously every week in combination with concurrent radiation. According to the most recent National Comprehensive Cancer Network (NCCN) guideline update, which is the most appropriate antiemetic regimen for nausea/vomiting prophylaxis for this patient to receive on treatment day 1?
A Ondansetron plus dexamethasone. B Fosaprepitant plus palonosetron. C Fosaprepitant plus prochlorperazine plus dexamethasone. D Palonosetron plus olanzapine plus lorazepam.
A - With the recent NCCN guideline update classifying carboplatin-containing regimens as highly emetogenic if the carboplatin dose has an AUC of 4 or greater and moderately emetogenic when the carboplatin dose has an AUC less than 4, this patient’s regimen would fall into the moderately emetogenic category (paclitaxel is low risk for emetogenicity). This would require standard treatment with a serotonin-3 antagonist and steroid, or the olanzapine regimen of olanzapine, palonosetron, and dexamethasone (Answer A is correct). A neurokinin-1 antagonist can potentially be added on in patients receiving moderately emetogenic regimens if they previously had treatment failure with a serotonin-3 antagonist and steroid or if they have significant risk factors, which this patient does not have noted (Answer B is incorrect). Prochlorperazine is not recommended by the NCCN guidelines as part of the day 1 prophylactic regimen and is instead recommended as an option for breakthrough nausea/vomiting (Answer C is incorrect). Lorazepam is recommended for anticipatory nausea and vomiting, which this patient does not have (Answer D is incorrect).
A new drug is being studied for managing acute heart failure. The impact of this new drug on the primary end point of death from cardiovascular causes was assessed by survival analysis using a Kaplan-Meier analysis. Which best describes the benefits of using survival analysis?
A All subjects required to enter the study at the same time. B Allows evaluation of the time course of treatment effect. C Does not allow data censoring because of loss of follow-up. D Only allows for assessment of mortality.
B - Survival analysis provides information relative to the temporal course of treatment effect (Answer B is correct), does not require subjects to enter the study at the same time (Answer A is incorrect), and does allow data censoring, one of its benefits (Answer C is incorrect). Answer D is incorrect because a survival analysis can be used for many different events such as time to disease progression or adverse events.
R.D. is an 87-year-old woman who was given a diagnosis of Alzheimer disease a year ago. After the diagnosis, she moved in with her daughter. Her daughter states that lately she wanders around the house continuously. She often changes clothes, cries out, and asks repetitive questions. She has a history of hypothyroidism, breast cancer, osteoarthritis in her right knee, and hypertension. Her current medication regimen includes donepezil 10 mg daily, which she has taken for the past 6 months; levothyroxine 50 mcg daily; hydrochlorothiazide 25 mg daily; and acetaminophen 650 mg three times daily. Which would be most appropriate for managing R.D.’s new behavioral symptoms?
A Initiate olanzapine 5 mg daily. B Initiate risperidone 0.5 mg twice daily. C Change the donepezil dosage to 23 mg once daily. D Change acetaminophen to a topical or systemic NSA
D - No evidence currently supports increasing the donepezil dose to 23 mg to manage the behavioral symptoms of dementia (Answer C is incorrect). Off-label use of atypical antipsychotic medications in patients with behavioral symptoms of dementia should be reserved for patients who pose a danger to themselves or others or who have hallucinations or delusions that are stressful to them (Answers A and B are incorrect). Changing acetaminophen to an NSAID to treat possible pain that could be causing the patient’s behavior should be tried before trying more aggressive strategies (Answer D is correct).
Your health system has identified the opportunity to improve immunizations for pregnant people admitted for delivery. Standardizing the administration of which immunization upon hospital admission would have the greatest impact on achieving Healthy People 2030 objectives?
A Hepatitis A. B Human papillomavirus (HPV). C Pneumococcal polysaccharide (PPSV23). D Tetanus, diphtheria, and pertussis (Tdap).
D - Healthy People 2030 has several objectives related to decreasing the prevalence of infectious disease through increased immunization rates, including pertussis in infants, HPV in adolescents, and acute hepatitis. When looking at the recommendations for use of certain vaccines during pregnancy, hepatitis A is only recommended during pregnancy if the patient is at risk of an infection or a severe outcome from infection while pregnant (Answer A is incorrect). The HPV is not recommended for use during pregnancy, and if the vaccine is to be administered upon hospital admission, the recipient would still be pregnant (Answer B is incorrect). A dose of PPSV23 is only recommended for those with certain conditions before age 65; therefore, routine administration is not recommended (Answer C is incorrect). The use of Tdap is recommended during each pregnancy to reduce the risk of pertussis in the infant; therefore, standardization of this vaccine upon hospital admission would likely have the largest impact on reaching the Healthy People 2030 goals (Answer D is correct).
Which benefit of automated dispensing cabinet technologies utilized by hospital systems best helps optimize pharmacy technician workload?
A Enhanced security of medications. B Increased measures to avert medication diversion. C Reduction in medication errors. D Enhanced inventory control.
D - Each option is a benefit of automatic dispensing cabinet technologies. Enhanced security of medications will help avert medication diversion, but these will not best optimize technician workload (Answers A and B are incorrect). Reduction in medication errors is a major benefit of automated dispensing cabinets, but this is primarily helpful to improve patient safety (Answer C is incorrect). Enhanced inventory control such as expansion of high-use medications and reduction of infrequently used medications decrease the volume and frequency of stocking and destocking medications, ultimately increasing efficiency of technician work (Answer D is correct).
Results from a randomized controlled trial showed a difference in 90-day mortality between the intervention group and the control group as 11% (p=0.03). Which statement is most accurate?
A The trial did not have adequate power. B The chance of a type II error was 3 in 100. C The chance of a type I error was 3 in 100. D There was no statistically significant difference between groups.
C - The typical a priori α (type I) rate is 5% (i.e., when the study was designed, the error rate was designed to be 5% or less). The actual type I error rate reported was 0.03 (3%), the p value (Answer C is correct). This difference was considered statistically significant because the p value was less than 0.05 (Answer D is incorrect). The study did have enough power because a statistically significant difference was observed (Answer A is incorrect). Similarly, a type II error was not made because this error has to do with not finding a difference when one truly exists (Answer B is incorrect).
A new oral anticoagulant was studied to evaluate its ability to prevent venous thromboembolism (VTE) in medically ill patients. In this randomized, double-blind, double-dummy trial, patients were randomized to receive enoxaparin 40 mg once daily during the hospitalization, with placebo also during the hospitalization and for 30 days after discharge, or the new oral anticoagulant during the hospitalization and for 30 days after discharge with placebo injections during the hospital stay. The primary efficacy end point was the rate of VTE at 30 days after discharge, and the primary safety end point was major bleeding. At the end of the trial, VTE had occurred less often with the use of extended prophylaxis with the new oral anticoagulant than with enoxaparin during hospitalization only (3.9% vs. 5.7%; RR 0.68 [95% CI, 0.53–0.88]). Major bleeding with extended prophylaxis was 0.7% compared with 0.5% with enoxaparin (RR 1.48; 95% CI, 0.77–2.84). Which is the most accurate interpretation of these data?
A Extended prophylaxis significantly reduced VTE and significantly increased major bleeding compared with enoxaparin. B Extended prophylaxis significantly reduced VTE without significantly increasing major bleeding compared with enoxaparin. C Extended prophylaxis increased major bleeding more than it reduced VTE compared with enoxaparin. D The reduction in VTE with extended prophylaxis was balanced against the increased risk of bleeding compared with enoxaparin.
B - The RR for the reduction in VTE with extended prophylaxis with the new oral anticoagulant compared with enoxaparin was less than 1, and the CI did not include 1, making the reduction in VTE statistically significant. Although the RR for major bleeding was greater than 1 for extended prophylaxis compared with enoxaparin, the CI included 1, making the difference not statistically significant. Therefore, Answer B is correct and Answer A is incorrect. The absolute reduction in VTE with extended prophylaxis compared with enoxaparin was 1.8, and the absolute increase in major bleeding was 0.2. Therefore, the increase in major bleeding was not balanced or more than the reduction in VTE with extended prophylaxis compared with enoxaparin, making Answers C and D incorrect.
C.T. is a 40-year-old woman who presents to the hospital with right leg pain and swelling. The erythema and swelling extend from her ankle to 2 inches below her knee. There is no purulent drainage and no leading edge. She also has chills. Her medical history includes asthma and obesity (diabetes in her father). She has a history of rash associated with penicillin use. Which is the best treatment for C.T.?
A Piperacillin/tazobactam 4.5 g intravenously every 8 hours plus vancomycin 15 mg/kg intravenously every 12 hours. B Ceftaroline 600 mg intravenously every 12 hours. C Doxycycline 100 mg intravenously every 12 hours. D Cefazolin 2 g intravenously every 8 hours.
D - This infection does not appear to be necrotizing fasciitis. Therefore, piperacillin/tazobactam plus vancomycin would provide too-broad antibacterial activity. There is no need for the gram-negative and anaerobic activity that piperacillin/tazobactam adds over vancomycin alone. Moreover, the patient is allergic to penicillins (Answer A is incorrect). This appears to be a cellulitis, but there is no indication from the patient’s symptoms and history that methicillin-resistant Staphylococcus aureus (MRSA) is involved. Therefore, ceftaroline is not necessary because its advantage over cefazolin is primarily in the addition of MRSA activity (Answer B is incorrect). Doxycycline has activity against Staphylococcus, including MRSA, but its streptococcal activity is too poor to use alone empirically in this situation, and there is no indication that MRSA is causing the infection (Answer C is incorrect). Cefazolin is the best choice in this situation because of its excellent activity against Staphylococcus (non-MRSA) and Streptococcus, the two most-suspected organisms causing cellulitis. Because the patient only describes her penicillin allergy as a rash, it is safe to try a cephalosporin at this time (Answer D is correct).
Which is the most appropriate candidate to receive the tetanus, diphtheria, pertussis (Tdap) vaccine?
A 64-year-old man who received his last Tdap vaccine at 53 years of age. B 32-year-old pregnant woman at 12 weeks' gestation. C 14-year-old female adolescent requiring wound management, who received a Tdap vaccine at 12 years of age. D 18-year-old man entering college, who received his first Tdap vaccine at 11 years of age.
A - In 2019, the ACIP updated its recommendations to allow Tdap administration when previously only tetanus, diphtheria (Td) was recommended, including decennial Td booster doses, tetanus prophylaxis for wound management, and catch-up immunizations for those 7 years and older with an incomplete or unknown vaccination history. Answer A is correct because this patient is currently overdue for his decennial booster dose. Although pregnant women should receive one dose of Tdap during each pregnancy, it should ideally be administered at 27–36 weeks’ gestation, making Answer B incorrect. Answer C is incorrect because Tdap or Td is indicated for wound management when more than 5 years have passed since the previous tetanus toxoid–containing vaccine dose. Answer D is incorrect because this patient is currently up to date with a routine adolescent Tdap dose and would only require a booster every 10 years.
A.J. is a 34-year-old man who has been HIV positive for the past 3 years. His current viral load is 200,000 copies/mL, and his CD4 count is 360 cells/mm3. Which is the best initial action to take for A.J.?
A Continue to monitor viral load and CD4 count, but no therapy is needed at this time. B Initiate tenofovir disoproxil fumarate 300 mg orally daily and emtricitabine 200 mg orally daily because his CD4 count is still above 200 cells/mm3. C Initiate dolutegravir 50 mg orally daily, emtricitabine 200 mg orally daily, and tenofovir disoproxil fumarate 300 mg orally daily. D Initiate darunavir/cobicistat 800/150 mg orally daily, abacavir 600 mg orally daily, and lamivudine 300 mg orally daily.
C - Current guidelines recommend that all patients with HIV infection be treated with highly active antiretroviral therapy, regardless of viral load or CD4 counts. Therefore, therapy should be initiated (Answer A is incorrect). Therapy should include a “recommended regimen for most patients.” Tenofovir disoproxil fumarate and emtricitabine without a third agent is not a recommended regimen (Answer B is incorrect). Darunavir/cobicistat with abacavir and lamivudine is recommended in certain clinical situations and is not considered a first-line regimen (Answer D is incorrect). Dolutegravir with emtricitabine/tenofovir disoproxil fumarate is correct. All of the current “recommended regimens” have an integrase inhibitor combined with two reverse transcriptase inhibitors (Answer C is correct).
Which of the following best depicts the organism that can be identified in a pharmacy through a Clinical Laboratory Improvement Amendments of 1988 (CLIA)–waived point-of-care test?
A Helicobacter pylori. B Staphylococcus aureus. C Hepatitis B virus. D Pseudomonas aeruginosa.
A - H. pylori is the only one of the listed organisms that can be identified through a CLIA-waived point-of-care test, making Answer A correct. Although hepatitis C virus can be identified through detected antibodies by a point-of-care test, this is not true for hepatitis B virus. Therefore, Answer C is incorrect. There are no CLIA-waived tests for S. aureus or P. aeruginosa, making Answers B and D incorrect.
Adenovirus, GAS, H.pulori, HCV, HIV, mononucleosis, Flu A and B, Borerrelia burgdoferi, RSV, Treponema pallidum, Trichomonas vaginalis
A 62-year-old man with a history of hypertension, type 2 diabetes, and dyslipidemia takes metformin 500 mg twice daily and atorvastatin 20 mg daily. His blood pressure at home and in the clinic averages 144/86 mm Hg. His SCr, urine albumin/creatinine, LDL, and A1C are 0.7 mg/dL, 89 mg/g, 68 mg/dL, and 6.9%, respectively. All laboratory measures are consistent with the values obtained 3 months ago. All additional laboratory values are within normal limits. Which recommendation is most appropriate for this patient at today’s visit?
A No change in therapy is needed. B Initiate enalapril. C Increase metformin. D Increase atorvastatin.
B - Initiating angiotensin-converting enzyme inhibitor therapy (Answer B) is the most appropriate answer because the patient’s blood pressure and urine albumin/creatinine remain elevated. Answer A is incorrect; doing nothing would place the patient at increased risk of damage from uncontrolled blood pressure. Answer C is incorrect because the patient’s A1C is adequately controlled. Increasing atorvastatin, Answer D, is also incorrect because the patient does not have a history of atherosclerotic cardiovascular disease and is appropriately receiving a moderate-intensity statin. In addition, insufficient information is provided to suggest the patient’s current atorvastatin dose is incorrect.
A health system has been notified of a drug shortage. Which is the first step when developing a drug shortage management plan?
A Assess the clinical impact of the drug shortage. B Determine the amount of product on hand. C Identify any medication safety risks associated with the shortage. D Plan for necessary changes in workflow.
B - There is a stepwise approach in developing a drug shortage management plan. All of the answer choices are appropriate steps; however, the first step is determining the amount of product on hand to approximate how long the product will last, in addition to when the next delivery will arrive, if known (Answer B is correct). Then, the clinical impact of the drug shortage is determined, which includes how essential the medication is with respect to saving lives or curing a disease, whether any alternatives are available, and which patients and clinicians will be affected by the shortage (Answer A). Next, safety risks associated with any alternative drugs or changes in formulation or strength are identified (Answer C). Overall workflow changes including storage and technology are addressed (Answer D). Finally, strategies on how the remaining product will be used are established
A 62-year-old woman with a history of diabetes and recent trauma is admitted to the hospital because of cellulitis/osteomyelitis caused by Staphylococcus aureus. Empirically, she is initiated on vancomycin at 1000 mg intravenously daily. You are consulted to monitor therapy. Which monitoring strategy is best?
A Obtain a peak concentration only to ensure concentrations reach at least 40 mg/L for adequate tissue perfusion and optimal efficacy. B Obtain a trough concentration only to ensure concentrations decrease below 15 mg/L during the dosing interval to avoid nephrotoxicity. C Obtain a trough concentration and potentially one other concentration to target an AUC/MIC ratio of 400–600. D Obtain peak and trough concentrations to optimize efficacy while minimizing toxicity.
C - Because vancomycin follows time-dependent killing, obtaining and assessing peak concentrations is not beneficial from an efficacy standpoint. Moreover, nephrotoxicity is most associated with higher trough concentrations, eliminating the need to monitor peak concentrations (Answers A and D are incorrect). Traditionally, trough concentrations have been monitored to maintain concentrations above the MIC of the organism while minimizing the incidence of nephrotoxicity. However, more recent data analyses support that achieving an AUC/MIC ratio of 400–600 both maximizes efficacy and minimizes toxicity (Answer C is correct). Calculations of AUC can be completed with one concentration and a Bayesian software program or two concentrations during the same dosing interval (Answer B is incorrect).
Which reference is best to find the most up-to-date information on a new drug for the treatment of heart failure with preserved ejection fraction?
A Journal of the American College of Cardiology. B American Hospital Formulary Service. C UpToDate. D Braunwald’s Heart Disease: A Textbook of Cardiovascular Medici
A - Answers B, C, and D all represent tertiary references, which are not the best references for investigating new information because they may have a several-year lag in the data provided. Despite titles like “UpToDate” or specific cardiology texts, Answers B, C, and D are incorrect. The Journal of the American College of Cardiology is the official journal of the American College of Cardiology, which is one of the leading cardiology organizations in the world. Answer A represents a primary literature source, which would provide the latest data in a well-respected cardiology journal. Therefore, Answer A is correct.
Primary - original research, immediate results, original research published in peer reviewed journals, dissertations, technical reports, conference proceedings
Secondary - Summarizes and synthesizes primary literature, literature review articles, books
Tertiary - summaries or condensed versions, references to primary or secondary sources, good place to look up facts or general overview. Textbooks, dictionaries, encyclopedias, handbooks
You are reviewing a manuscript that reports the results from a study of the effect of pharmacist involvement on a general medical team. The authors find that teams that include pharmacists make fewer errors, and the estimates are presented as the mean number of errors plus or minus the standard error of the mean (SEM). In your review, you comment on the authors’ choice of the SEM as their measure of variability. Which best describes the authors’ choice?
A Appropriate because it estimates the spread of the data. B Inappropriate because it estimates the variability of the individual observations, not of the means estimated from repeated iterations of the study. C Appropriate because it overestimates the variability of the data. D Inappropriate because it estimates the variability of the means estimated from repeated iterations of the study, not of the individual observations.
D - Answer D is correct because SEM does not measure the spread (Answer A is incorrect) or the variability (Answers B and C are incorrect) of data. The SEM is estimated by dividing the standard deviation by the square root of the sample size. The SEM alone does not measure the variability of data and often underestimates the variability of a data set (again making Answer C incorrect). That leaves Answer D, which correctly describes the purpose of the SEM.
M.F. is a 45-year-old man with recently diagnosed Parkinson disease. Currently, his symptoms consist of a tremor in his right hand. Other than taking hydrochlorothiazide for hypertension, he is in good health. At this time, which would be the best treatment for M.F.?
A Carbidopa/levodopa. B Ropinirole. C Entacapone. D Trihexyphenidyl.
D - In Parkinson disease, the more effective drugs should be reserved for later in treatment as the disease progresses. Ropinirole (Answer B) and carbidopa/levodopa (Answer A) are highly effective for Parkinson disease and treat many of the symptoms in addition to tremor. Most treatment schemes recommend waiting until the Parkinson symptoms progress. Trihexyphenidyl (Answer D) is effective when the patient is only symptomatic for a tremor. Entacapone (Answer C) helps extend the effectiveness of carbidopa/levodopa through a dosing interval but is not highly effective on its own.
The relationship between a newly marketed drug and standard of care for impact on asthma exacerbations leading to hospitalizations was studied in a prospective cohort study. The results of the study showed a risk ratio (RR) of 2.0 with a 95% CI of 1.5–2.5.
Question 65
Which one of the following statements is most consistent with these results?
A The new drug is associated with no change in exacerbations. B The new drug is associated with a decrease in exacerbations. C The new drug is associated with an increase in exacerbations. D More information is needed to decide whether the new drug is associated with any changes in exacerbations.
C - The key to this question is the 95% CI. When interpreting an RR, an equal risk would be reported as an RR of 1. For an RR greater than 1, as in this case, the risk of developing an exacerbation would be higher in the patients receiving the new drug than in the patients not receiving this drug. Furthermore, because the CI does not cross or include 1.0, the point estimate in this study (2.0) is statistically significant (Answer C). Both Answer A (CI would need to include 1) and Answer B (RR would need to be less than 1 and CI could not include 1 for this to be correct) are incorrect. Answer D is incorrect because the information given is adequate to answer this question.
The relationship between a newly marketed drug and standard of care for impact on asthma exacerbations leading to hospitalizations was studied in a prospective cohort study. The results of the study showed a risk ratio (RR) of 2.0 with a 95% CI of 1.5–2.5.
Question 66
Given the information provided, which statement best represents the statistical significance of the reported RR?
A Because the p value was not presented, it cannot be determined whether there is a statistically significant difference between the groups. B The RR is statistically significant at p<0.05. C The RR is statistically significant at p<0.01. D The RR is not statistically significant.
B - The CI does not cross or include 1.0, indicating that the point estimate in this study (2.0) is statistically significant (Answer B is correct). Answer A is incorrect because enough information is present to answer this question, and a p value is not required. According to the question, the RR and CI information allows the appropriate answer to this question. The p value of less than 0.05 corresponds to an α value less than 0.05 and is thus statistically significant. This p value also corresponds with the 95% CI. For Answer C to be correct, the 99% CI would need to be determined and the same evaluation of the CI made. Furthermore, because the CI does not cross or include 1.0, the point estimate in this study (2.0) is statistically significant (Answer D is incorrect).
T.T. is a nurse who works at your institution. While obtaining blood from a patient known to be HIV positive, she accidentally stuck herself. The needle went through her glove and broke the skin on her hand. The patient’s most recent CD4 count was 220 cells/mm3, and her most recent viral load was 50,000 copies/mL. Which therapy is best for T.T.?
A No drug therapy is needed – just disinfect the area well. B Offer emtricitabine 200 mg orally daily for 2 weeks. C Offer emtricitabine 200 mg and tenofovir alafenamide 25 mg orally daily for 3 months. D Offer emtricitabine 200 mg orally daily, tenofovir disoproxil fumarate 300 mg orally daily, and raltegravir 400 mg orally twice daily for 4 weeks.
D - Because this was a significant needlestick from a known HIV-positive patient, T.T. should be offered drug therapy (Answer A is incorrect). Although it is generally recommended that postexposure prophylaxis last 4 weeks (not 2 weeks), emtricitabine monotherapy is inappropriate for occupational postexposure prophylaxis (Answer B is incorrect). Emtricitabine plus tenofovir alafenamide is indicated for preexposure prophylaxis, but a regimen with only two agents is not recommended for postexposure prophylaxis. Moreover, 3 months is too long a duration (Answer C is incorrect). The most appropriate prophylactic regimen is raltegravir with tenofovir disoproxil fumarate/emtricitabine for 4 weeks (Answer D is correct).
R.N. is a 68-year-old man with a history of hypertension and atrial fibrillation who is receiving digoxin. Verapamil is initiated for further ventricular rate control. Which best predicts what will happen to his digoxin concentration if verapamil is initiated?
A Will increase significantly because more digoxin will be absorbed. B Will decrease significantly because more digoxin will be removed during the first pass through the liver. C Will decrease significantly because more digoxin will be secreted in the kidneys. D Will increase significantly because less digoxin will distribute into peripheral tissues.
A - Digoxin concentrations increase significantly when verapamil is added . This is because verapamil blocks P-glycoprotein, which normally decreases the amount of digoxin that is absorbed in the GI tract (by pumping it back into the GI lumen) (Answer A is correct). Verapamil also blocks P-glycoprotein in the kidneys, which decreases renal secretion of digoxin, also increasing its concentration (Answer C is incorrect). These effects are not the result of any effect within the liver (Answer B is incorrect). Verapamil does not affect the distribution of digoxin into peripheral tissues (Answer D is incorrect).
A recent randomized controlled trial of a new drug was designed with 85% power to detect a 20% difference in the primary end point of blood pressure reduction. The trial targeted 500 subjects for enrollment on the basis of the sample size calculation. The secondary end point, occurrence of adverse drug events, was also studied. The power associated with this secondary end point in detecting a 6% difference in the occurrence of adverse drug events was 80%. Because of the increased costs associated with the trial, the investigators could only enroll 400 subjects. In the discussion section of the paper describing the study findings, the authors commented that because of lower sample size, the power associated with detecting a 20% difference in blood pressure was still an acceptable 80%. Which is the most critical problem caused by the reduced sample size?
A Randomization procedure was suboptimal. B Study was underpowered to detect a difference in the primary end point (blood pressure reduction). C Study was underpowered to detect rare adverse events. D Study was underpowered to detect a difference in the secondary end point (occurrence of adverse drug events).
D - In the discussion section, the authors correctly pointed out that even though they could not recruit the appropriate number of subjects, 80% power was obtained, which is considered an acceptable level of power in the biomedical literature for the primary end point. Answer D is correct because the authors could not maintain adequate power for comparison of the secondary end point. Because the original power for that comparison was 80%, it would have fallen below the 80% level typically considered optimal. No information is presented to conclude that the randomization was suboptimal (Answer A is incorrect). Answer B is incorrect because the power is adequate. Answer C is incorrect because this size of trial could not identify rare adverse events, regardless of subject enrollment.
The results of a randomized controlled trial between two drug treatments for depression show no difference in clinical response between the two treatment groups (p=0.25). Clinical response was the primary end point of the study. The study authors report that they subsequently analyzed subgroups on the basis of the following: subjects with past episodes of depressive episodes, subjects with a family history of depression, and subjects who were women. The authors found a significant difference in response between the two treatments (p<0.01) for men versus women. Which statement is most accurate regarding the results of the subgroup analysis?
A The finding of a significant difference between men and women in this study should be tested in a prospective study. B Post hoc subgroup analyses are generally useful for showing differences between patient subgroups. C Statistical adjustments are not needed when comparing the results of each subgroup. D Men and women are expected to respond differently to depression pharmacotherapy.
A - Subgroup analyses are vulnerable to many problems in interpretation (Answer B is incorrect). In general, subgroup analyses not specified at the beginning of the study are likely to have issues of power and the absence of correction to the statistical analysis to account for multiple subgroup comparisons (Answer C is incorrect). Findings from post hoc analyses are best viewed as hypothesis generating (Answer A is correct), similar to other observational studies, not as answers to the questions themselves (Answer D is incorrect). There are some exceptions to this discussion.
A.J. is a 45-year-old woman with multiple sclerosis (MS). She has been treated with glatiramer and ß-interferon, which have slowed the disease progression. In the clinic today, she notes increased difficulty walking. Her walking speed has slowed greatly. She asks whether anything can help with her walking. Which medication would best help with walking?
A Amantadine. B Fingolimod. C Natalizumab. D Dalfampridine.
D - Fingolimod (Answer B) and natalizumab (Answer C) are disease-modifying treatments for MS; however, although these agents slow disease progression in MS, they do not help with acute treatment of symptoms. Amantadine (Answer A) relieves fatigue symptoms associated with MS, but not difficulties with ambulation. Dalfampridine (Answer D) is specifically indicated for difficulty walking with MS.